Finding area between polar curves

Click For Summary
The discussion centers on calculating the area between the polar curves of a lemniscate defined by r^2 = 2sin(2θ) and a circle with r = 1. The user initially set the limits of integration at π/12 and (5π)/12 but received a different area approximation from a classmate, leading to confusion about the correct method. The correct approach involves integrating the area of the lemniscate and subtracting the area of the circle segment, which is a third of a circle, resulting in a total area of approximately 0.68485. The key error identified was in the integration setup, particularly the need to correctly apply the formula and limits. The final suggested method involves integrating the radial functions and properly accounting for the areas involved.
Sentience
Messages
77
Reaction score
0

Homework Statement



Find the area of the region inside the lemniscate r^2 = 2sin(2\theta) and outside the circle r = 1

It sucks because I wish I could post a graph, but the graph on my calculator looks like a circle around the origin with radius 1, with an infinity symbol going diagonally through it. (starting in quad 3 and pointing into quad 1.)

I believe the area they are after is the two ends of the lemniscate that don't quite fit inside the circle. (quads 1 and 3)

Homework Equations



1/2 * integral of (f(\theta)^2 - g(\theta)^2)

with your limits of integration being the angles where the two polar functions intersect.

The Attempt at a Solution



I set the equations equal to each other, and get the angles pi/12 and (5*pi)/12 as the limits of integration. Now, based on what the graph looks like, and the fact that I'm only after the ends of the lemniscate, it seems that:

2 * 1/2 * integral of ( 2sin(2\theta)^2 - (1)^2) with my limits of integration being pi/12 and (5 * pi)/12 would do the trick. I get an answer of pi/3. (approx. 1.05)

However, a classmate gave me the answer, an approximation, of .682. He said he went over it with the teacher, and I'm pretty sure he's right. Any tips on where I went wrong?
 
Physics news on Phys.org
You said your classmate's figure was approximate and I do get approximately:rolleyes: that, 0.68485.

Your approach as expressed in the integral appears perfectly OK.

Clue to where you have gone wrong is that your π/3 actually is the second part of your integral that you have to subtract from the first. I understand your g(θ) = 1. You don't need to do any integration of this part, it is just a third of a circle. Oh that's 2π/3. Well you work out the details. :biggrin: The pi factor should alert you to where error is because a trigonometric function of any fraction of pi wouldn't itself be any multiple of π.

Personally I would just integrate r2/2 between your limits, the whole shaded area in fig I will try to put up shortly, and subtract π/3 from that.
 
Last edited:
2euoq4j.jpg


Hope you can make out the circles of radii 1 and 2 there.

In the formula should have been cos 2θ not cosθ
 
You did it right, I think you probably pressed the wrong buttons on your calculator.

Here is how I did it

2\int_{\frac{\pi}{12}}^{\frac{5\pi}{12}}} \int_{1}^{\sqrt{2\sin2\theta}} r dr d\theta
 
Question: A clock's minute hand has length 4 and its hour hand has length 3. What is the distance between the tips at the moment when it is increasing most rapidly?(Putnam Exam Question) Answer: Making assumption that both the hands moves at constant angular velocities, the answer is ## \sqrt{7} .## But don't you think this assumption is somewhat doubtful and wrong?

Similar threads

  • · Replies 2 ·
Replies
2
Views
2K
  • · Replies 19 ·
Replies
19
Views
2K
  • · Replies 20 ·
Replies
20
Views
2K
  • · Replies 10 ·
Replies
10
Views
2K
Replies
3
Views
2K
  • · Replies 6 ·
Replies
6
Views
2K
  • · Replies 3 ·
Replies
3
Views
2K
Replies
2
Views
2K
  • · Replies 1 ·
Replies
1
Views
2K
  • · Replies 2 ·
Replies
2
Views
1K